[ 3 / biz / cgl / ck / diy / fa / ic / jp / lit / sci / vr / vt ] [ index / top / reports ] [ become a patron ] [ status ]
2023-11: Warosu is now out of extended maintenance.

/sci/ - Science & Math


View post   

File: 10 KB, 707x153, ss (2014-05-15 at 02.07.40).png [View same] [iqdb] [saucenao] [google]
6535974 No.6535974 [Reply] [Original]

I hate asking /sci/ for hw help, but I don't have much experience with proving things.

I need help proving a summation inequality, I easily showed it was true for n=1, and assumed it was true for n=k, now the inductive step showing it is true for n=k+1, I have no clue how to go about it.

>> No.6535985

1. Prove it's true for 1.
2. Prove it's true for 2.

Congratulations, you've proved it's true for n+1.

>> No.6535994

>>6535985

fuck off

>> No.6536007

>>6535974
well, add <div class="math"> \frac{1}{(n+1) ^2} </div> to both sides
then show that the right side of that inequality is less than or equal to the "n+1 version" of the right side

>> No.6536138

The left side will never reach 2 it will keep getting closer and as it reaches closer to 2 the other side moves towards 2 so it will always be smaller

>> No.6536158
File: 1.56 MB, 320x165, 1399966647790.gif [View same] [iqdb] [saucenao] [google]
6536158

>>6536138
silly rabbit
thats not how proof works

>> No.6536164

show that it is true for n = 1 (trivial). Then, assume it is true for n=k. Adding 1/(k+1)^2 to both sides will give you the inequality you need, as 1/(k+1)^2 < 1/k.

>> No.6536226

>>6535974


<span class="math"> \sum_{1}^{n+1}\frac{1}{k} =\sum_{1}^{n}\frac{1}{k}+\frac{1}{k+1}<2-\frac{1}{n}+\frac{1}{n+1} [/spoiler]

<span class="math"> =2-\frac{1}{n+1}\frac{(n+1)^2}{n^2}<2-\frac{1}{n+1} [/spoiler]

For n>0.
P(n)=>P(N=1)

>> No.6536230

>>6536226
Fuck. I missed a bunch of squares out when i latexd it. But that's the gist

>> No.6536233

>>6536226
Corrected

<span class="math">\sum_{1}^{n+1}\frac{1}{k^2} =\sum_{1}^{n}\frac{1}{k^2}+\frac{1}{(k+1)^2}<2-\frac{1}{n}+\frac{1}{(n+1)^2}[/spoiler]

<span class="math">=2-\frac{1}{n+1}\frac{(n+1)^2}{n^2}<2-\frac{1}{n+1}[/spoiler]